Category Archives: Toán Chuyên

Đề và đáp án thi chọn đội dự tuyển lớp 10 năm 2012 – 2013

Bài 1. Cho $a,b$ là hai số thực thoả mãn $a+b\ge 0$. Chứng minh rằng:

$$\left(\dfrac{a^2+b^2}{2}\right)^3\ge 4(a^3+b^3)(ab-a-b).$$

Bài 2. Tìm tất cả các số nguyên dương $m,n$ để $\dfrac{5mn+5m}{3m^2+2n^2}$ là số nguyên.

Bài 3.  Cho tập hợp $X={1,2,\ldots,2n-1}$ gồm $2n-1$ số tự nhiên $(n\ge 2)$. Tô màu ít nhất $n-1$ phần tử của $X$ với điều kiện sau: nếu $a,b\in X$ (không nhất thiết phân biệt) được tô màu thì $a+b$ cũng được tô màu, miễn là $a+b\in X$. Gọi $S$ là tổng tất cả các số không được tô màu của $X$.

a/Chứng minh rằng $S\le n^2$.

b/Chỉ ra tất cả các phép tô màu sao cho $S=n^2$.

Bài 4. Cho đường tròn $(O)$ và dây cung $AB$ cố định khác đường kính. Gọi $C$ là điểm chính giữa cung lớn $AB$. Đường thẳng $d$ thay đổi qua $C$ cắt tiếp tuyến tại $A$ và tiếp tuyến tại $B$ của $(O)$ lần lượt tại $D,E$. Gọi $Q$ là giao điểm của $AE$ và $BD$. Chứng minh rằng đường thẳng $PQ$ luôn đi qua một điểm cố định khi $d$ thay đổi.

Giải

Bài 1. Ta xét các trường hợp sau:

  •  Nếu $ab<0$, ta có vế trái dương và vế phải âm nên bất đẳng thức đúng.
  •  Nếu $ab \ge 0$, kết hợp với $a+b \ge 0$, ta suy ra $a,b \ge 0.$

Áp dụng lần lượt các đánh giá $4xy \le (x+y)^2$ và $2xy \le x^2 + y^2$ thì:

$$\begin{align*} 4(a^3+b^3)(ab-a-b) & = 4(a+b)(ab-a-b)(a^2-ab+b^2) \\ & \leq a^2b^2(a^2-ab+b^2) \\ & \leq \dfrac{ab(a^2+b^2)^2}{4} \end{align*}$$

Mà ta có:

$$\dfrac{(a^2+b^2)^3}{8}=\dfrac{(a^2+b^2)^2}{4}.\dfrac{a^2+b^2}{2}\geq \dfrac{ab(a^2+b^2)^2}{4}.$$

Từ hai đánh giá trên, ta có đpcm.

Bài 2.

Đặt $k=\dfrac{5mn+5m}{3m^2 + 2n^2} \in \mathbb{N}^*$. Suy ra

$$3km^2 – 5(n+1)m + 2kn^2 = 0$$ là một phương trình theo ẩn $m$ với

$$\Delta = 25(n+1)^2 – 24k^2n^2 = (25-24k^2)n^2 + 50n + 25 \ge 0. (*)$$

Xét các trường hợp sau:

  • Nếu $k>1$, ta có:

$\Delta _1′ = 625 – 25\left( {25 – 24{k^2}} \right) = 600{k^2} > 0$, mà $25 – 24k^2 < 0$.

Suy ra bất phương trình $(*)$ có nghiệm khi $n \le \dfrac{25+10k\sqrt{6}}{24k^2-25}< 2$ (dễ dàng chứng minh).

Vì thế nên $n=1$ (do $n \in \mathbb{N^{*}}$). Ta có:

$$ \begin{aligned} k= \dfrac{10m}{3m^2 + 2} \in \mathbb{N^{*}} & \Rightarrow \dfrac{30m^2}{3m^2 + 2} \in \mathbb{N^{*}} \Rightarrow \dfrac{-20}{3m^2 + 2} \in \mathbb{N^{*}} \\ & \Rightarrow 3m^2 +2 \in \left\{ {2;5;10;20} \right\} \text{ vì } 3m^2+2 \ge 2, \forall m \\ & \Rightarrow m=1 \text{ do } m \in \mathbb{N^{*}}. \end{aligned} $$

Thử lại ta nhận $(m;n)=(1;1)$

  •  Nếu $k=1$ thì $\Delta = n^2 + 50n +25 = x^2$ ($x \in \mathbb{N}$) nên suy ra $$(n+x+25)(n-x+25) = 600.$$

Từ đây với lưu ý $n+x+25 > n-x+25 > 0, n+x+25 > 25$ ta có $$n \in \left\{ {126;52;28;10;6} \right\}.$$ Thay vào phương trình đầu, ta tìm được  $$(m;n)=(9;6),(5;10),(32;28),(32;52),(81;126).$$

Bài 3.

(a) Rõ ràng nếu $1$ được tô thì tất cả các số cũng sẽ được tô, kéo theo $S=0 \le n^2$, thỏa mãn. Do đó, ta chỉ cần xét $1$ không được tô. Gọi các số được tô là $$1 < a_1 < a_2 < \ldots < a_m \le 2n-1,$$

trong đó $m \ge n-1$. Ta sẽ chứng minh rằng với mọi $k$ mà $1 \le k \le m/2$ thì

$$a_k + a_{m-k+1} \ge 2n.$$

Giả sử ngược lại rằng $a_k+a_{m-k+1} <2n$ thì tổng hai số trên phải là số được tô màu. Do đó, nó phải thuộc tập hợp

$$Q = \left\{ {{a_{m – k + 2}};{a_{m – k + 3}};\ldots;{a_m}} \right\}.$$

Mặt khác lại xét chỉ số $i < k$ thì rõ ràng do dãy đang xét là tăng nên ta cũng có tổng ${a_i} + {a_{m – k + 1}}$ thuộc tập hợp $Q$ ở trên. Suy ra $|Q| \ge k,$ mâu thuẫn vì rõ ràng $Q$ chỉ có $k-1$ phần tử. Vì thế nên ta phải có $a_k + a_{m-k+1} \ge 2n.$

Đến đây, ta có ${a_k} + {a_{m – k + 1}} \ge 2n$ với mọi $k \in \left\{ {1;2;3;\ldots;m} \right\}$ nên

$$\sum\limits_{i = 1}^m {{a_i} = \frac{1}{2}} \sum\limits_{i = 1}^m {({a_i} + {a_{m – i + 1}}) \geqslant n(n – 1)}, \text{ suy ra }$$

$$S = \sum\limits_{i = 1}^{2n – 1} i – \sum\limits_{i = 1}^m {{a_i} \leqslant n(2n – 1) – n(n – 1) = {n^2}}.$$

(b) Để có $S=n^2$ thì dấu bằng xảy ra ở tất cả các đánh giá trên, tức là ta tô được đúng $m=n-1$ số và $a_k+a_{n-k}=2n$ với mọi $1 \le k \le n-1.$

Ta có $(2{{a}_{1}},{{a}_{1}}+{{a}_{2}},{{a}_{1}}+{{a}_{3}},\ldots ,{{a}_{1}}+{{a}_{n-2}})$ là một hoán vị của các số $({{a}_{2}},{{a}_{3}},\ldots ,{{a}_{n-1}}).$

Do tính tăng của hai dãy này nên ta có $$2{{a}_{1}}={{a}_{2}},{{a}_{1}}+{{a}_{2}}={{a}_{3}},\ldots ,{{a}_{1}}+{{a}_{n-2}}={{a}_{n-1}}.$$ Vì thế nên ${{a}_{k}}=k{{a}_{1}}$ với mọi $1\le k\le n-1.$ Mà $2n={{a}_{1}}+{{a}_{n-1}}=n{{a}_{1}}$ nên ta có ${{a}_{1}}=2,$ từ đây tìm được các tô duy nhất là $(2,4,6,\ldots ,2n-2)$ thỏa mãn đề bài.

Bài 4.

Giả sử $AD\cap BE=T,AB\cap DE=I$ và $TQ$ cắt $DE,AB$ lần lượt ở $X,S.$ Khi đó dễ thấy rằng

$(IX,DE)=(IS,AB)=-1.$

Mà $PI$ đi qua trung điểm cung lớn $AB$ của $(O)$ nên $PI$ là phân giác ngoài, kéo theo $PS$ là phân giác trong nên nó đi qua $N$ là trung điểm cung nhỏ $AB$ của $(O)$.

Gọi $M$ là trung điểm $AB.$ Theo tính chất phương tích thì $TN\cdot TC=T{{A}^{2}}=T{{B}^{2}}=TM\cdot TO$, mà $O$ là trung điểm $CN$ nên theo hệ thức Maclaurin thì $(TM,NC)=-1.$

Không có mô tả.

 

Lại có $(TQ,XS)=-1$ nên chùm $P(XS,QT)=-1$, mà $PX$ đi qua $C,$ $PS$ đi qua $N$ nên ta phải có $PQ$ đi qua $M$ là điểm cố định.

Nhận xét: Bài toán có thể xử lý theo hướng tự nhiên hơn bằng cách dùng định lý Ceva sin. Từ kết quả trên, ta còn thấy được rằng nếu lấy $CQ$ cắt $AB$ ở $K$ thì $PK$ là đối trung của tam giác $PAB,$ kéo theo $P,K,T$ thẳng hàng.

Đáp án đề thi chọn đội dự tuyển lớp 10 năm 2016 – 2017

Bài 1: Cho $x,y,z$ là các số thực dương thoả mãn $x+y+z=1$. Chứng minh rằng:

$$\dfrac{x^4}{x^3+y^2+z^2}+\dfrac{y^4}{y^3+z^2+x^2}+\dfrac{z^4}{z^3+x^2+y^2}\ge \dfrac{1}{7}.$$

Bài 2: Tìm tất cả các hàm số $f:\mathbb N^* \rightarrow \mathbb N^*$ thoả mãn đồng thời các điều kiện:

i/ $f(mn)=f(m)f(n)\ \forall m,n \in \mathbb N^*$.

ii/ $f(m)+f(n)$ chia hết cho $m+n$ $\forall m,n \in \mathbb N^*$.

iii/ $f(2017)=2017^3$.

Bài 3. Cho đường tròn $(O)$ và dây cung $AB$ cố định. $C$ là một điểm thay đổi trên cung lớn $AB$ sao cho tam giác $ABC$ nhọn. Gọi $I,I_a,I_b$ lần lượt là tâm đường tròn nội tiếp, tâm đường tròn bàng tiếp $\angle BAC$ và $\angle ABC$ của tam giác $ABC$.

a/ Gọi $M$ đối xứng với $I$ qua $O$. Chứng minh rằng tam giác $MI_{a}I_{b}$ cân.

b/ Gọi $H,K$ lần lượt là hình chiếu của $I_a,I_b$ trên $OI$. Đường thẳng qua $H$ vuông góc với $BI_a$ và đường thẳng qua $K$ vuông góc với $AI_b$ cắt nhau tại $P$. Chứng minh rằng $P$ thuộc một đường cố định khi $C$ thay đổi.

Bài 4. Cho $S$ là tập hợp khác rỗng và $A_1,A_2,\ldots,A_m\ (m\ge 2)$ là $m$ tập con của $S$. Gọi $\mathcal T$ là tập hợp gồm tất cả các tập hợp $A_i\Delta A_j\ (1\le i,j \le m$). Chứng minh rằng $|\mathcal T| \ge m$.

(Ký hiệu $A\Delta B=(A\backslash B)\cup (B\backslash A)$ là hiệu đối xứng của hai tập hợp $A,B$).

Giải

Bài 1.

Theo bất đẳng thức Cauchy-Schwarz, ta có

$$ \sum \dfrac{x^4}{x^3+y^2+z^2} \ge \dfrac{ \left( x^2+y^2+z^2 \right)^2}{ x^3+y^3+z^3+2 \left( x^2+y^2+z^2 \right)} $$

Cần chứng minh $\dfrac{ \left( x^2+y^2+z^2 \right)^2}{ x^3+y^3+z^3+2 \left( x^2+y^2+z^2 \right)} \ge \dfrac{1}{7} $ hay

$$7 \left( x^2+y^2+z^2 \right)^2 \ge x^3+y^3+z^3+2 \left( x^2+y^2+z^2 \right).$$ Ta có ${{(xy+yz+zx)}^{2}}\ge 3xyz(x+y+z)=3xyz$ và

$${{x}^{3}}+{{y}^{3}}+{{z}^{2}}-3xyz=(x+y+z)({{x}^{2}}+{{y}^{2}}+{{z}^{2}}-xy-yz-zx)$$ nên ${{x}^{3}}+{{y}^{3}}+{{z}^{3}}=3xyz+1-3(xy+yz+zx)\le {{(xy+yz+zx)}^{2}}+1-3(xy+yz+zx).$

Đặt $q=xy+yz+zx$ thì vì ${{(x+y+z)}^{2}}\ge 3(xy+yz+zx)$ nên $q\le \frac{1}{3}.$ Ta đưa về

$$7{{(1-2q)}^{2}}\ge {{q}^{2}}+1-3q+2(1-2q)$$ hay

$$(1-3q)(4-9q)\ge 0.$$

Do $q\le \frac{1}{3}$ nên $q\le \frac{4}{9}$ và bất đẳng thức trên là đúng. Vậy ta có đpcm.

Bài 2.

Nhận xét rằng vai trò của số $2017$ trong bài toán là không cần thiết cho nên ta sẽ giải bài toán khi thay $2017$ bởi số nguyên dương $p$ bất kỳ. Từ điều kiện đầu tiên, ta có được $f(p^k)=p^{3k}$ với $k$ là số nguyên dương bất kỳ.

Trong điều kiện thứ hai, thay $n$ bởi $m$, ta có $f(m)$ là bội của $m$ với mỗi $m$ nguyên dương nên ta đặt $f(m)=m.g(m)$ ($g:\mathbb{N^{*}}\rightarrow \mathbb{N^{*}}$). Khi đó ta có các điều kiện sau:

i/ $g(mn)=g(m).g(n) \forall m,n \in\mathbb{N^{*}}$

ii/ $mg(m)+ng(n)$ là bội của $m+n$.

iii/ $g(p^{n})=p^{2n} \forall n\in \mathbb{N^{*}}$.

Đặt $h(m)=g(m)-m^2$ ($h:\mathbb{N^{*}}\rightarrow \mathbb{Z}$) và thay $n$ bởi $p^n$ tại ii), ta có $m.h(m)$ là bội của $m+p^n$. Chọn $n$ đủ lớn thì $h(m)=0$ với mỗi $m$ hay $f(m)=m^3$ với mỗi $m$ nguyên dương. Thử lại thoả mãn.

Vậy $f(m)=m^3$ là nghiệm hàm duy nhất.

Bài 3.

(a) Trước hết, ta có một kết quả quen thuộc sau.

Bổ đề: Gọi $A_1$, $B_1$ lần lượt là điểm chính giữa các cung $BC$, $AC$ không chứa $A$, $B$ của $(O)$. Khi đó $A$, $I$, $A_1$, $I_a$ thẳng hàng và $A_1$ là trung điểm của $II_a$. Tương tự đối với $B$, $I$, $B_1$, $I_b$.

Trở lại bài toán, theo bổ đề, phép vị tự tâm $I$, tỉ số $2$ biến $\Delta OA_1B_1$ thành $\Delta MI_aI_b$, do đó tam giác này cân tại $M$.

Mở ảnh

(b) Ta thực hiện chuyển đổi mô hình. Gọi $I_a$ là tâm bàng tiếp góc $A$ của tam giác $ABC$ thì $(O)$ chính là đường tròn Euler của tam giác $I_aI_bI_c$. Xét bổ đề sau:

Bổ đề: Cho tam giác $ABC$ có đường thẳng $d$ đi qua tâm ngoại tiếp $O$. Gọi $D,E,F$ lần lượt là hình chiếu của $A,B,C$ lên $d$. Chứng minh rằng đường thẳng qua $D,E,F$ vuông góc với $BC,CA,AB$ đồng quy trên đường tròn $Euler$ của tam giác $ABC$.

Gọi $l$ là đường thẳng đi qua trực tâm $H$ của tam giác $ABC$ và vuông góc với $d$. Gọi $S$ là điểm anti-Steiner của $l$. $J$ là điểm đối xứng của $S$ qua $BC$ và $X$ là giao điểm của $SJ$ và $(O)$. $K$ là điểm đối xứng với $H$ qua $BC$. \medskip

Ta có: $$\angle AXS =\angle AKS=\angle KHJ$$ suy ra $HJ \parallel AX$. Do đó, $D$ nằm trên $AX$ hay $D$ là trung điểm $AX$. Suy ra đường thẳng qua $D$ vuông góc với $BC$ đi qua trung điểm $I$ của $SH$ và nằm trên đường tròn $Euler$ của tam giác $ABC$. \medskip

Trở lại bài toán, ta áp dụng bổ đề trên cho đường thẳng $OI$ đi qua tâm đường tròn $(I_aI_bI_c)$ thì dễ dàng có $P \in (O).$

Bài 4.

Ta sẽ chứng minh quy nạp theo $m$ cho điều này. Trước hết ta phát biểu bổ đề: $A\Delta B=A\Delta C$ thì $B=C$.

Giả sử $B\neq C$, khi đó không giảm tổng quát giả sử giả sử $a$ là phần tử thỏa $a\in B$ và $a\notin C$. Ta có hai trường hợp như sau:

  •  Nếu $a\in A$ khi đó $a\notin (A\setminus B),a\notin (B\setminus A)\Rightarrow a\notin A\Delta B$. Nhưng lại có $a\in (A\setminus C)$ nên suy ra $a\in A\Delta C$ nên $A\Delta B\neq A\Delta C$, vô lý.
  •  Nếu $a\notin A$ thì chứng minh tương tự suy ra $a\notin A\Delta C$ và $a\in A\Delta B$ nên suy ra $A\Delta B\neq A\Delta C$. Như vậy ta suy ra $B=C$.

Bây giờ ta sẽ quy nạp theo $m$. Với $m=1$ thì ta có một tập thuộc $T$ là tập rỗng. Với $m=2$ và hai tập $A,B$ thì ta có hai tập thuộc $T$ là tập rỗng và $A\Delta B$ thỏa. Như vậy giả thiết đúng với $m=1,2$.

Giả sử giả thiết đúng với $m=k$ thì ta chứng minh nó đúng với $m=k+1$. Xét $m+1$ tập $A_1,A_2,\ldots,A_{m+1}$. Nếu với $m$ tập $A_1,A_2,\ldots,A_m$ mà số lượng tập tạo thành không nhỏ hơn $m+1$ thì khi đó ta thêm vào một tập $A_{m+1}$ thì giả thiết vẫn đúng. Do đó ta chỉ xét cho trường hợp $|T|=m$.

Khi đó, nếu ta thêm vào một tập $A_{m+1}$ thì ta sẽ thêm vào tập $T$ các tập hợp $A_{m+1}\Delta A_1,\ldots,A_{m+1}\Delta A_{m+1}$. Nếu các tập này trùng với $m$ tập đã có trong $T$ thì do $|T|=m$ nên theo nguyên lý Dirichlet tồn tại $i,j,1\leq i<j\leq m+1$ để $A_{m+1}\Delta A_i=A_{m+1}\Delta A_j$ và theo bổ đề ta có $A_i=A_j$, vô lý. Vậy trong $m+1$ tập đó chắc chắn có một tập khác với các tập trong $T$ và số phần tử của $T$ tăng lên ít nhất một đơn vị, tức là $|T|\geq m+1$.

Vậy giả thiết quy nạp là đúng và ta có đpcm.

Đáp án thi chọn đội tuyển trường PTNK năm 2014

Đề bài

Ngày thi thứ nhất

Bài 1. Cho $a,b,c > 0$ thỏa mãn điều kiện $(a+1)(b+1)(c+1)=1+4abc$.
Chứng mình rằng ta có bất đẳng thức $a+b+c\le 1+abc.$
Bài 2. Cho tập hợp $A=\left\{ {{n}^{3}}-4n+15|n\in \mathbb{N} \right\}.$ Tìm tất cả các phần tử $a\in A$ thỏa mãn đồng thời hai điều kiện sau đây:

i) $a$ là số chẵn.
ii) Nếu $a_1,a_2$ là các ước số của $\dfrac{a}{2}$ với $a_1,a_2>1$ thì $\gcd (a_1,a_2)>1$.

Bài 3. Tìm tất cả các hàm số $f:\mathbb N^* \rightarrow \mathbb N^*$ thỏa mãn:
$$f \left(\dfrac{f(n)}{n} \right)=n^2 \ \forall n\in \mathbb N^*.$$
Bài 4. Cho tam giác $ABC$ nội tiếp $(O)$, có $B,C$ cố định và $A$ thay đổi trên $(O).$ Ký hiệu $(I)$ là đường tròn nội tiếp tam giác $ABC.$ Gọi $({{O}_{1}})$ là đường tròn qua $A,B$ và tiếp xúc với đường tròn $(I)$ tại $E.$ Gọi $({{O}_{2}})$ là đường tròn qua $A,C$ và tiếp xúc với đường tròn $(I)$ tại $F$. Đường phân giác trong của góc $\angle AEB$ cắt $({{O}_{1}})$ tại $M$ và đường phân giác trong của góc $\angle AFC$ cắt $({{O}_{2}})$ tại $N.$

a) Chứng minh rằng tứ giác $EFMN$ nội tiếp.
b) Gọi $J$ là giao điểm của $EM$ và $FN$. Chứng minh rằng đường thẳng $IJ$ luôn đi qua một điểm cố định.

Ngày thi thứ hai

Bài 5. Cho dãy số $({{x}_{n}})$ bởi $x_0=1,x_1=2014$ và $x_{n+1}=\sqrt[3]{x_nx_{n-1}^2}\ \forall n\in \mathbb{N}^*.$

a) Chứng minh rằng dãy số $(x_n)$ có giới hạn hữu hạn và tìm giới hạn đó.
b) Với mỗi $n\ge 2$, hãy tìm số nguyên dương $k$ nhỏ nhất sao cho $a=x_n^k$ là một số nguyên. Chứng minh rằng khi đó $a$ không thể viết được dưới dạng tổng các lũy thừa bậc ba của hai số tự nhiên.

Bài 6. Cho $X$ là tập hợp gồm $19$ phần tử.

a) Chứng minh rằng tồn tại ít nhất $2600$ tập con $7$ phần tử của $X$ sao cho với hai tập con $A,B$ bất kỳ trong số $2600$ tập con đó, ta có $\left| A \cap B \right|\le 5.$
b)  Xét một họ $\Omega $ gồm $k$ tập con có $7$ phần tử của $X$. Một tập $A\subset X$ được gọi là một cận trên của $\Omega $ nếu như $\left| A \right|=8$ và tồn tại một tập con $F$ của họ $\Omega $ sao cho $F\subset A$. Gọi $d$ là số tập con cận trên của họ $\Omega$. Chứng minh rằng $d\ge \frac{3}{2}k.$

Bài 7. Cho tam giác $ABC$ không cân. Gọi $I$ là trung điểm $BC$. Đường tròn $(I)$ tâm $I$ đi qua $A$ cắt $AB,AC$ lần lượt tại $M,N.$ Giả sử $MI,NI$ cắt $(I)$ tại $P,Q$. Gọi $K$ là giao điểm của $PQ$ với tiếp tuyến tại $A$ của $(I)$. Chứng minh rằng $K$ thuộc đường thẳng $BC.$
Bài 8. Tìm số nguyên dương $n$ lớn nhất thỏa mãn các điều kiện sau:

a) $n$ không chia hết cho $3$.
b) Bảng vuông $n\times n$ không thể được phủ kín bằng $1$ quân tetramino $1\times 4$ và các quân trimino $1\times 3$. Trong phép phủ, các quân tetramino và trimino được phép quay dọc nhưng không được phép chườm lên nhau hoặc nằm ra ngoài bảng vuông.

Hết

Lời giải

Bài 1.

Điều kiện đã cho viết thành $ab+bc+ca+a+b+c=3abc$.
Chia hai vế cho $abc$ rồi đặt $a=\frac{1}{x},b=\frac{1}{y},c=\frac{1}{z}$, ta có
$xy+yz+zx+x+y+z=3.$ \medskip

Bất đẳng thức đã cho có thể viết thành
$$xy+yz+zx-xyz\le 1 \text{ hay } x+y+z+xyz\ge 2. $$
Theo bất đẳng thức Schur thì $${{(x+y+z)}^{3}}+9xyz\ge 4(xy+yz+zx)(x+y+z). $$
Đặt $m=x+y+z,n=xy+yz+zx$ thì $m+n=3$ và
$$xyz\ge \frac{4mn-{{m}^{3}}}{9}. $$
Ta sẽ chứng minh rằng
$$m+\frac{4mn-{{m}^{3}}}{9}\ge 2\Leftrightarrow {{m}^{3}}+4{{m}^{2}}-21m+18\le 0 $$ hay
$(m-2)({{m}^{2}}+6m-9)\le 0.$
Chú ý rằng ${{m}^{2}}\ge 3n$ nên
$${{m}^{2}}\ge 3(3-m)\Leftrightarrow {{m}^{3}}+3m\ge 9. $$
Do đó ${{m}^{2}}+6m-9\ge 0.$
Ta xét các trường hợp

Nếu $m>2$ thì $x+y+z>2$ nên hiển nhiên bất đẳng thức cần chứng minh là đúng.
Nếu $m\le 2$ thì $m-2\le 0$ nên ta cũng có $(m-2)({{m}^{2}}+6m-9)\le 0.$

Vậy trong mọi trường hợp, ta luôn có đpcm.

Bài 2.

Ta thấy rằng $a=n^3-4n+15$ chẵn nên $n^3+15$ chẵn hay $n$ lẻ. Đặt $n=2k+1$ với $k \in \mathbb{N}$. Ta có
$\begin{aligned}
a={{n}^{3}}-4n+15 & =(n+3)({{n}^{3}}-3n+15) \\
& =(2k+4)(4{{k}^{2}}-2k+3) \\
\end{aligned} $
nên $\frac{a}{2}=(k+2)(4{{k}^{2}}-2k+3)$.
Điều kiện ii) cho thấy rằng $\frac{a}{2}$ phải là lũy thừa của một số nguyên tố, vì nếu nó có hai ước nguyên tố trở lên, đặt là $p,q$ thì chọn $x=p,y=q$, ta có $x,y>1$ nhưng $\gcd (x,y)=1,$ không thỏa. \medskip

Vì $(4{{k}^{2}}-2k+3)-(k+2)=4{{k}^{2}}-3k+1>0$ với mọi $k\in \mathbb{N}$. Do đó, ta phải có $k+2|4{{k}^{2}}-2k+3$. Suy ra
$\frac{4{{k}^{2}}-2k+3}{k+2}=4k-10+\frac{23}{k+2}\in \mathbb{Z}. $
Do đó $k+2\in \{1,23\}$ vì $k+2>0.$ Ta xét các trường hợp

Nếu $k+2=1$ thì $k=-1$ hay $n=2k+1=-1<0$, không thỏa.
Nếu $k+2=23$ thì $k=21$ hay $n=43$, tính được $\frac{a}{2}=3\cdot {{5}^{2}}\cdot {{23}^{2}}$, cũng không thỏa.

Vậy không tồn tại số $a$ nào thỏa mãn.

Bài 3.

Với $n\in \mathbb{N}*$, ta thấy rằng nếu $n=1$ thì $f(f(1))=1$. \medskip

Nếu $n>1$ thì gọi $p$ là một ước nguyên tố bất kỳ của $n$. \medskip

Vì $\frac{f(n)}{n}\in \mathbb{N}*$ nên $n|f(n)$. Đặt $a={{v}_{p}}(n),b={{v}_{p}}\left( f(n) \right)$ thì trước hết, ta có $a\le b.$ \medskip

Từ $f\left( \frac{f(n)}{n} \right)={{n}^{2}}$, ta suy ra rằng $\left. \frac{f(n)}{n} \right|{{n}^{2}}$ hay $f(n)|{{n}^{3}}$, tức là $b\le 3a.$ \medskip

Trong biểu thức đã cho, thay $n\to \frac{f(n)}{n}$ thì
$f\left( \frac{f\left( \frac{f(n)}{n} \right)}{\frac{f(n)}{n}} \right)={{\left( \frac{f(n)}{n} \right)}^{2}}\Leftrightarrow f\left( \frac{{{n}^{3}}}{f(n)} \right)={{\left( \frac{f(n)}{n} \right)}^{2}}$
Do đó, ta phải có $\left. {{\left( \frac{f(n)}{n} \right)}^{2}} \right|\frac{{{n}^{3}}}{f(n)}\Leftrightarrow \left. {{f}^{3}}(n) \right|{{n}^{5}} \text{ nên } 3b \le 5a.$$ Sau đó lại tiếp tục thay $n$ trong biểu thức đã cho bởi $\frac{{{n}^{5}}}{{{f}^{3}}(n)}$ và cứ như thế, ta xây dựng được hai dãy hệ số của $a,b$ như sau
${{u}_{0}}={{v}_{0}}=1,{{u}_{1}}=3,{{v}_{1}}=1 \text{ và } $
${{u}_{k+1}}=2{{u}_{k-1}}+{{v}_{k}},{{v}_{k+1}}=2{{v}_{k-1}}+{{u}_{k}} \text{ với } k\ge 1. $
Khi đó $\frac{{{v}_{2k}}}{{{u}_{2k}}}\le \frac{b}{a}\le \frac{{{u}_{2k+1}}}{{{v}_{2k+1}}}. $
Biến đổi công thức của hai dãy, ta có ${{u}_{n+2}}=5{{u}_{n}}-4{{u}_{n-2}},{{v}_{n+2}}=5{{v}_{n}}-4{{v}_{n-2}}$ và cả hai dãy đều có phương trình đặc trưng là ${{t}^{2}}-5t+4=0$. Ngoài ra, dãy chẵn và dãy lẻ trong mỗi dãy đều độc lập với nhau.

Ta có ${{u}_{0}}=1,{{u}_{2}}=3,{{v}_{0}}=1,{{v}_{2}}=5$ nên
\[{{u}_{2k}}=\frac{13+2\cdot {{16}^{k}}}{15},{{v}_{2k}}=\frac{11+4\cdot {{16}^{k}}}{15},k\ge 1. \] Từ đó, dễ dàng tính được $\lim \frac{{{u}_{2k+1}}}{{{v}_{2k+1}}}=2$. \medskip

Một cách tương tự, ta tính được $\lim \frac{{{u}_{2k}}}{{{v}_{2k}}}=\frac{1}{2}$. Do đó, số $\frac{b}{a}$ bị kẹp ở giữa và là số nguyên nên chỉ có thể là $\frac{b}{a}=2\Leftrightarrow b=2a.$ \medskip

Rõ ràng tập hợp ước nguyên tố của $n$ và $f(n)$ là giống nhau. Hơn nữa, với một ước nguyên tố cụ thể thì số mũ trong $f(n)$ gấp đôi số mũ trong $n.$ Suy ra $f(n)={{n}^{2}}, \forall n>1.$ \medskip

Tiếp theo, giả sử $f(1)=n>1$ thì ta có $f(f(1))=1$ nên $f(n)=1,$ mâu thuẫn. Vì thế nên chỉ có thể $f(1)=1.$ \medskip

Vậy tất cả các hàm thỏa mãn là $f(n)={{n}^{2}},\forall n\in \mathbb{N}^*$.

Bài 4.

(a) Trước hết, ta thấy rằng ${{O}_{1}},I,E$ thẳng hàng và ${{O}_{2}},I,F$ thẳng hàng. \medskip

Vì $M$ là trung điểm cung $AB$ của $({{O}_{1}})$ nên ${{O}_{1}}M$ là trung trực của $AB$, suy ra $O\in {{O}_{1}}M.$ Tương tự, ta cũng có $O\in {{O}_{1}}N.$ \medskip

Gọi $P,Q$ lần lượt là tiếp điểm của $(I)$ với $AB,AC.$ \medskip

Vì $IP\parallel {{O}_{1}}M$ (cùng vuông góc với $AB$) nên $\angle M{{O}_{1}}E=\angle PIE$.
Hơn nữa, các tam giác ${{O}_{1}}ME,IPE$ đều cân với đỉnh là ${{O}_{1}},I$ nên suy ra chúng đồng dạng, tức là
$\angle IEP=\angle {{O}_{1}}EM$ hay $E,P,M$ thẳng hàng. Tương tự thì $F,Q,N$ cũng thẳng hàng. \medskip

Vì ta đã có $E,F,P,Q$ cùng thuộc đường tròn $(I)$ nên để có $E,F,M,N$ cùng thuộc một đường tròn thì $\angle EMN=\angle EFN=\angle EPQ$ hay $MN\parallel PQ.$ \medskip

Mặt khác, $AI\bot PQ$ nên ta cần có $AI\bot MN.$ \medskip

Thật vậy, sử dụng phương tích với đường tròn $(I)$ ta có
\[M{{A}^{2}}-N{{A}^{2}}=MP\cdot ME-NQ\cdot NF=M{{I}^{2}}-N{{I}^{2}} \] nên theo định lý bốn điểm thì $AI\bot MN$, từ đó ta có đpcm. \medskip

(b) Vì $PQ\parallel MN,OM\parallel IP$ nên dễ dàng có $\angle IPQ=\angle OMN$. Tương tự $\angle IPQ=\angle ONM.$ \medskip

Do đó, hai tam giác $IPQ,OMN$ đồng dạng với nhau, tức là $$\frac{IP}{OM}=\frac{PQ}{MN}.$$
Ngoài ra, $$\frac{JP}{JM}=\frac{IP}{OM},$$ kết hợp với $\angle JPI=\angle JMO$, ta có hai tam giác $JPI,JMO$ đồng dạng, dẫn đến $$\angle PJI=\angle MJO.$$

Từ đây suy ra $I,J,O$ thẳng hàng hay $IJ$ luôn đi qua điểm $O$ cố định.

Bài 5.

(a) Đặt $u_n=\log_{2014}(x_n)$ thì ta thu được dãy $(u_n)$ như sau
$\left\{\begin{matrix} u_0=0,u_1=1\\ u_{n+1}=\dfrac{1}{3}u_n+\dfrac{2}{3}u_{n-1} \end{matrix}\right. $
Từ đó tìm được
$u_n=\dfrac{3}{5}-\dfrac{3}{5} \cdot \left ( \dfrac{-2}{3} \right )^n$
Suy ra $\underset{n\rightarrow +\infty }{\lim}u_n=\dfrac{3}{5}$ nên ta có được
$\underset{n\rightarrow +\infty }{\lim} x_n=\underset{n\rightarrow +\infty }{\lim}(2014^{u_n})=2014^{3/5} $
(b) Ta thấy rằng để có $(x_n)^k$ là một số nguyên thì $\dfrac{3k(3^{n}-(-2)^n)}{5 \cdot 3^n}\in \mathbb{Z}$ nguyên.
Ta xét các trường hợp

Nếu $n$ lẻ thì $3^n-(-2)^n=3^n+2^n\;\vdots\; 5$. Vì $\gcd\left ( \dfrac{3^n+2^n}{5},3^n \right )=1$ nên ta được $3^n\mid 3k$ nên $k$ nhỏ nhất thỏa mãn điều này là $k=3^{n-1}$.
Nếu $n$ chẵn thì $3^n-2^n\equiv (-2)^n-2^n=0 \pmod{5}$ và tương tự, ta cũng tìm được $k=3^{n-1}$.

Do đó số $k$ nhỏ nhất cần tìm là $k=3^{n-1}$.
Tiếp theo, ta sẽ chứng minh rằng phương trình sau không có nghiệm tự nhiên
$a^3+b^3=2014^n \Leftrightarrow (a+b)(a^2-ab+b^2)=2014^n $

Gọi ${{n}_{0}}$ số nguyên dương nhỏ nhất sao cho tồn tại $a,b\in {{\mathbb{Z}}^{+}}$ để ${{a}^{3}}+{{b}^{3}}={{2014}^{{{n}_{0}}}}$.
Dễ thấy ${{n}_{0}}=1$ không thỏa nên ta chỉ xét ${{n}_{0}}\ge 2.$
Ta xét các trường hợp

Nếu $\gcd (a+b,{{a}^{2}}-ab+{{b}^{2}})=1$ thì dễ thấy ${{(a-b)}^{2}}\ge 1$. Khi đó
\[{{a}^{2}}-ab+{{b}^{2}}\ge a+b>\sqrt{{{a}^{2}}-ab+{{b}^{2}}}. \] Vì $2014=2\cdot 19\cdot 53$ nên chỉ có thể xảy ra \[a+b={{19}^{{{n}_{0}}}},{{a}^{2}}-ab+{{b}^{2}}={{106}^{{{n}_{0}}}}. \] Ngoài ra ${{(a+b)}^{2}}\le 4({{a}^{2}}-ab+{{b}^{2}})$ nên ta phải có ${{361}^{{{n}_{0}}}}\le 4\cdot {{106}^{{{n}_{0}}}}$. Đánh giá này sai khi ${{n}_{0}}\ge 2$ nên trường hợp này không thỏa.
Nếu $\gcd (a+b,{{a}^{2}}-ab+{{b}^{2}})>1$ thì chẳng hạn
\[a+b={{2}^{x}}u,{{a}^{2}}-ab+{{b}^{2}}={{2}^{y}}v \text{ với } \gcd (u,2)=\gcd (v,2)=1.\] Các trường hợp còn lại chứng minh tương tự.
Ngoài ra $uv={{1007}^{{{n}_{0}}}},x+y={{n}_{0}}. $
Chú ý rằng ${{(a+b)}^{2}}-({{a}^{2}}-ab+{{b}^{2}})=3ab$ nên $3ab$ cũng chẵn, tức là cả hai số $a,b$ đều chẵn (vì nếu không thì ${{a}^{3}}+{{b}^{3}}$ lẻ).

Từ đây dễ dàng chứng minh được $3{{v}_{2}}(a)=3{{v}_{2}}(b)={{n}_{0}}$, ta đưa về
${{{x}’}^{3}}+{{{y}’}^{3}}={{1007}^{{{n}_{0}}}}$.
Cứ như thế, ta được $2014|a,2014|b$ nên phương trình sau cũng có nghiệm nguyên dương
${{\left( \frac{a}{2014} \right)}^{3}}+{{\left( \frac{b}{2014} \right)}^{3}}={{2014}^{{{n}_{0}}-3}}. $
Điều này mâu thuẫn với các chọn ${{n}_{0}}$ nên phương trình trên vô nghiệm. Các trường hợp còn lại tương tự.
\end{enumerate}
Ta có đpcm.

Bài 6.

(a) Không mất tính tổng quát, ta có thể giả sử $X$ là tập hợp $19$ số nguyên dương đầu tiên.
Gọi $X(k)$ là tập hợp tất cả các tập con có $7$ phần tử của $X$ và tổng các phần tử của nó chia $19$ dư $k$. \medskip

Khi đó, dễ thấy rằng $\left| X(0) \right|+\left| X(1) \right|+\cdots +\left| X(18) \right|$ chính là số tập con có $7$ phần tử tùy ý của $X$ và là $C_{19}^{7}.$ \medskip

Ta thấy rằng hai tập hợp $A,B\in X(k)$ tùy ý đều thỏa mãn đề bài. \medskip

Thật vậy, \medskip

Giả sử $\left| A\cap B \right|=6$ (không thể có $\left| A\cap B \right|=7$ vì khi đó hai tập hợp trùng nhau). Đặt
$A=\{{{a}_{1}},{{a}_{2}},{{a}_{3}},{{a}_{4}},{{a}_{5}},{{a}_{6}},x\},B=\{{{a}_{1}},{{a}_{2}},{{a}_{3}},{{a}_{4}},{{a}_{5}},{{a}_{6}},y\}$
thì \[\sum\limits_{i=1}^{6}{{{a}_{i}}}+x\equiv \sum\limits_{i=1}^{6}{{{a}_{i}}}+y\equiv k\pmod{19}\] nên $x\equiv y\pmod{19}$. Suy ra $x=y$, mâu thuẫn.
Đến đây, dễ thấy rằng \[\underset{0\le k\le 18}{\mathop{\max }}\,\left\{ \left| X(k) \right| \right\}\ge \frac{C_{19}^{7}}{19}=2652>2600.\]Ta có đpcm. \medskip

(b) Xét một tập hợp $F$ thuộc họ $\Omega $. Vì $\left| X\backslash F \right|=19-7=12$ nên có tất cả $12$ tập hợp $A\subset X$ với $\left| A \right|=8$ và $F\subset A.$ \medskip

Ngược lại, ứng với một tập hợp $A$ là một cận trên của họ $\Omega $, có không quá $8$ tập $F$ trong họ $\Omega $ sao cho $F\subset A.$ Do đó $d\ge \frac{12}{8}k$ hay $d\ge \frac{3}{2}k.$ \medskip

Đẳng thức xảy ra khi họ $\Omega $ là tập hợp tất cả các tập con có $7$ phần tử của $X$.

Bài 7.

Không mất tính tổng quát, giả sử $AB<AC.$ \medskip

Kẻ đường kính $AJ$ của đường tròn $(I).$ Khi đó, dễ thấy tứ giác $ABJC$ và $ANJQ$ là các hình bình hành nên $JB\parallel AC,JQ\parallel AN$ dẫn đến $J,Q,B$ thẳng hàng. Tương tự $J,P,C$ thẳng hàng. \medskip

Gọi $H$ là hình chiếu của $A$ lên $BC$ thì tứ giác $AQBH$ nội tiếp. \medskip

Suy ra
\[\angle QHB=\angle QAB=\angle QAM=\angle QPM=\angle QPI \] nên tứ giác $PQHI$ cũng nội tiếp.
Gọi $(O)$ là đường tròn ngoại tiếp tam giác $ABC$ thì dễ thấy đường tròn $(AHI)$ tiếp xúc với $(O)$ tại $A.$

Xét ba đường tròn $(O),(AHI),(PQHI)$ thì

Trục đẳng phương của $(O),(AHI)$ là tiếp tuyến của $(O)$ tại $A$.
Trục đẳng phương của $(O),(PQHI)$ là $PQ$.
Trục đẳng phương của $(PQHI),(AHI)$ là $HI.$

Do đó, $K$ chính là tâm đẳng phương của ba đường tròn nên $K\in HI$ hay $K,B,C$ thẳng hàng.

Bài 8.

Ta sẽ chứng minh $n = 5$ là giá trị lớn nhất cần tìm. \medskip

Ta nhận thấy rằng nếu $n = 3k+1, k \ge 1$ thì ta luôn phủ được bảng vuông $n \times n$ bằng cách phủ hàng đầu tiên bằng $1$ quân tetramino kích thước $1 \times 4$ (ta sẽ gọi tắt là tetramino) và $k-1$ quân trimino kích thước $1 \times 3$ (ta sẽ gọi tắt là trimino). Các cột còn lại có chiều dài $3k$ có thể phủ được bằng các quân trimino (xoay dọc lại). \medskip

Ta chứng minh rằng nếu $n = 3k+2, k \ge 2$ thì bảng vuông $n \times n$ cũng phủ được. Cách phủ với $n = 8$ được minh họa như sau

Dễ dàng thấy rằng với $k \ge 3$ thì ta có thể thu được cách phủ cho bảng vuông $n \times n$ bằng cách phủ phần hình vuông $8 \times 8$ ở góc trên bên trái như trên, phần còn lại gồm $1 $ hình chữ nhật kích thước $3(k-2) \times (3k+2)$ và 1 hình chữ nhật kích thước $8 \times 3(k-2)$ phủ được bằng các quân trimino.

Bây giờ ta chứng minh bảng vuông $5 \times 5$ không thể phủ được bằng 1 quân tetramino và 7 quân trimino.

Trước hết ta chứng minh bổ đề: Nếu bảng vuông $5 \times 5$ có thể phủ được bằng một hình vuông $1 \times 1$, ta gọi là unomino và $8$ quân trimino thì quân unomino $1 \times 1$ phải phủ ô trung tâm. \medskip

Thật vậy, \medskip

Ta đánh số các ô của bảng vuông $5 \times 5$ như hình vẽ

Ta thấy rằng một quân trimino luôn phủ đúng một ô mang số $1,$ một ô mang số $2$ và một ô mang số $3.$ Vì số các số $2$ bằng $9,$ còn số các số $1$ và $3$ bằng $8$ nên nếu phép phủ ở đề bài thực hiện được thì quân unomino phải phủ một ô mang số $2.$ \medskip

Mặt khác, ta có thể đánh số bảng vuông $5 \times 5$ bằng một cách khác

Các tính chất nói ở trên vẫn đúng cho cách đánh số này, tuy nhiên ở đây số số $1$ là $9$, còn số số $2$ và $3$ là $8.$ Do đó, một lần nữa ta kết luận quân unomino phải phủ một ô mang số $1.$ \medskip

Giao hai điều kiện cần nói trên lại, ta thấy với một cách phủ hợp lệ thì quân unomino phải phủ ô trung tâm. \medskip

Quay trở lại với vấn đề phủ bảng vuông $5 \times 5$ bằng $1$ quân tetramino và $7$ quân trimino. Nếu tồn tại một cách phủ như thế thì cắt quân tetramino thành $1$ quân unomino và $1$ quân trimino, ta thu được một phép phủ bảng vuông $5 \times 5$ bằng $1$ quân unomino và $8$ quân trimino. \medskip

Theo bổ đề thì quân unomino phải nằm ở ô trung tâm, nghĩa là một đầu của quân tetramino phải nằm ở ô trung tâm, mâu thuẫn (vì khi đó quân tetramino sẽ bị lòi ra ngoài bảng vuông). \medskip

Với những lý luận ở trên, ta kết luận $n = 5$ là giá trị lớn nhất cần tìm.